HS328

अब Quizwiz के साथ अपने होमवर्क और परीक्षाओं को एस करें!

Two years ago, Isabella purchased 100 shares of CVC, Inc., for $15,000. Unfortunately, the value of the shares has dropped to $10,000. Isabella's daughter, Shea, was heading off to college, and Isabella was tired of waiting for a return on the stock. Isabella gave the stock to Shea when it was worth $10,000 to help fund Shea's education. If Shea sells the shares for $12,000 three months after the transfer, what is the amount and character of her gain or loss? A) $0 B) $2,000 short-term capital gain C) $3,000 long-term capital loss D) $3,000 short-term capital loss

A) $0 Isabella gave loss property to her daughter. When a taxpayer transfers property with a loss to someone else, and the recipient sells the property for an amount between the donor's adjusted basis and the fair market value of the stock on the date of the gift, no gain or loss is recognized. In this example, the no-gain, no-loss corridor is from $10,000 (the fair market value of the stock on the date of the gift) to $15,000 (the adjusted basis in the hands of the donor). If Shea sold the stock for any amount between $10 and $15 thousand dollars, she is not required to recognize any gain, and she is prohibited from recognizing any loss on the transaction. Since there is no gain or loss, there is no need to categorize the tax result as either long or short term.

The North Equity Fund has a beta of 1.67 and a standard deviation of 22.6 percent. It has returned 13.8 percent during the past year when the return on one-year treasury bills has been 3.6 percent. The Sharpe Ratio of the North Equity Fund is closest to A) 0.45 B) 0.61 C) 6.11 D) 8.26

A) 0.45 Sharpe Ratio = (Portfolio Return - Risk-Free Rate) ÷ (Standard Deviation)Sharpe Ratio = (13.8 − 3.6) ÷ 22.6 = 0.45

A portfolio has the following stocks: Weight Expected Return Stock 1 50% Weight 20% Stock 2 20% Weight 7% Stock 3 30% Weight 6% What is the weighted return of the portfolio? A) 13.2% B) 16.0% C) 14.0% D) 10.0%

A) 13.2% (0.50 × 20) + (0.20 × 7) + (0.30 × 6) = 13.2

Which of the following statements about mutual fund loads is correct? A) A back end load may be charged when an investor redeems shares of a mutual fund shortly after buying them. B) A no-load mutual fund will have either no front-end load or no back-end load. C) Loads are considered transaction costs and do not appear in the fund's prospectus. D) Back-end loads are included in a fund's annual expense ratio until the load is paid off.

A) A back end load may be charged when an investor redeems shares of a mutual fund shortly after buying them. Back-end loads are typically charged when an investor redeems shares of a mutual fund shortly after buying those shares. Statement (B) is incorrect because a no-load fund will have neither a front-end nor a back-end load. Statement (C) is incorrect because loads are not considered transaction costs and will appear in a fund's prospectus. Statement (D) is incorrect because back-end loads are not included in the expense ratio and will never be paid unless the investor redeems shares.

Which of the following would most likely be classified as a primary market transaction? A) A mortgage-backed security (MBS) purchased from an institution that originates the mortgage loans B) Equity shares purchased on the floor of the New York Stock Exchange C) A corporate bond purchased from an endowment fund that is liquidating D) Shares of preferred stock that are purchased from a dealer

A) A mortgage-backed security (MBS) purchased from an institution that originates the mortgage loans Primary market transactions are those in which the financial securities are created, which would include the creation of an MBS by the originating institution. The equity and preferred shares and the corporate bond are examples of secondary market transactions.

Brooks has been following Amazon stock for years. All forms of analysis leads him to firmly believe the stock is overpriced. Based on this conclusion, he shorts 1,000 shares of Amazon. Which of the following positions would help hedge his short? A) Buy 10 Amazon call options. B) Sell 10 Amazon call options. C) Buy 10 Amazon put options. D) Sell 10 Amazon put options.

A) Buy 10 Amazon call options Brooks has a short stock position, which means he borrowed the shares that he sold short. If the stock price increases, his loss potential is unlimited. The long call allows the Amazon stock to be bought at a fixed price, limiting his risk.

Colin purchased Apple stock earlier this year. The stock has increased significantly. While Colin likes the company's long-term prospects, he is concerned about a rapid price drop. Which of the following strategies would be protect him from such a price drop? A) Buy Apple puts. B) Sell Apple puts. C) Buy Apple calls. D) Sell Apple calls.

A) Buy Apple puts A put option protects against drops in Apple stock price.

Which type of mutual fund share generally converts to an "A share" after a period of time? A) Class B shares B) Class C shares C) Both B and C shares D) Neither B nor C shares

A) Class B shares Most Class B shares have a conversion feature that allows the shares to convert to Class A after a specified time period. Only in certain cases may Class C shares convert to A shares.

Which of the following regarding futures contracts is least accurate? A) Futures contracts are less liquid than forward contracts. B) Futures contracts are marked to market. C) Futures contracts are traded on a regulated exchange. D) Futures contracts allow more delivery options than forward contracts.

A) Futures contracts are less liquid than forward contracts. Futures contracts are more liquid than forward contracts, because they are traded on organized exchanges and are standardized. Forward contracts are customized to the parties involved in the transaction.

Gene and Jean are married taxpayers who have a high income and are in the top marginal tax bracket. They own the home they live in. The home has greatly appreciated in value since he bought it and they are concerned about the taxes they will owe if they sell it. Which of the following statements best describe the tax-savings opportunities available to them? A) If they have lived in the home at least 2 of the past 5 years, they will not pay taxes on the first $500,000 of gain. B) They can depreciate the value of the home, saving on taxes today; and, when they sell the home, they can recapture the depreciation at a lower tax rate than the top marginal rate. C) If they perform a Section 1031 exchange, they may be able to defer or avoid taxation on the gain. D) If they gift the home to a relative, that relative will receive a step-up-in-basis; the relative can then sell the home without recognizing any gain.

A) If they have lived in the home at least 2 of the past 5 years, they will not pay taxes on the first $500,000 of gain. As a married couple, Gene and Jean can exclude the first $500,000 of gain from their taxable income (assuming they lived in the home at least 2 of the past 5 years). Statements (B) and (C) are incorrect because these rules apply to business property but not personal property. Statement (D) is incorrect because the step-up-in-basis rules apply to inherited property but not gifted property.

Which of the following statements concerning a unit investment trust is correct? A) It has an unmanaged portfolio. B) It usually consists of common stocks. C) It is most appropriate for young people. D) It requires constant management attention.

A) It has an unmanaged portfolio. Option (B) is incorrect as UITs generally consist of bonds. Option (C) is incorrect as UITs are better for generating income, not the typical objective of young people. Option (D) is incorrect as the fund is generally passive.

Index funds are most notable for which one of the following features? A) Low management fees B) Outperformance of the market C) Liquidation during open-market hours D) Trading on exchanges

A) Low management fees Index fund managers do not perform significant levels of research on security selection and allocation, which tends to reduce management fees. Index funds are among the funds with the lowest expenses and fees. Option (B) is incorrect because index funds do not tend to outperform the market; rather, they tend to mimic the market. Option (C) is incorrect because ETFs, not index funds, may be liquidated during open-market hours. Option (D) is incorrect because closed-ended funds and ETFs are traded on exchanges; index funds are bought and sold directly from the fund manager.

One of your clients wants to build a portfolio characterized by high liquidity and very low risk. This client does not mind if the portfolio's returns are low. Given these specifications, you might recommend a portfolio consisting primarily of which asset class? A) Money market instruments B) Municipal bonds C) Corporate bonds D) Equities

A) Money market instruments Money market instruments are the least risky asset class with the lowest return. Municipal and corporate bonds, as fixed income securities, have a moderate risk and return profile. Equities tend to offer the highest return as well as highest risk.

Primary commodities likely include: A) Oil extracted from wells B) Refined gasoline C) Steel produced from iron ore D) "2 by 4s" made in a lumber yard

A) Oil extracted from wells Primary commodities are those that are considered natural resources that can be mined or extracted and used without much processing needed for consumption. Examples include oil, gold, and wheat. When a commodity must undergo a significant process to be able to be consumed, it is known as a secondary commodity. Examples include refined gas, lumber, and steel.

Preferred stock is a hybrid security. In which ways in preferred stock like common stock? A) Representation of ownership interest B) Guaranteed dividend payments C) Premiums attached to its par value D) Payment of interest

A) Representation of ownership interest Preferred stock represents an equity ownership position. Preferred shareholders typically receive a fixed dividend (not interest) payment, although no dividend payments are ever guaranteed. Par value is independent of the notion of a hybrid security

Examples of income stocks are least likely to include stock categories: A) Tech B) Blue chip C) Utility D) Financial institutions

A) Tech Tech stocks typically are high growth firms that require any cash flows to be reinvested back into the business, leaving little for the payment of dividends. Blue chips, utilities, and financial institutions generally pay dividends, often even high dividends.

According to the Black-Scholes option pricing model, what will happen to the average cost of call options if interest rates increase? A) The average cost will increase. B) The average cost will decrease. C) For interest sensitive stocks, but not for other stocks, the average cost will increase. D) For interest sensitive stocks, but not for other stocks, the average cost will decrease.

A) The average cost will increase. According to the Black-Scholes option pricing model, interest rate increases will increase the average cost of all call options. Higher interest rates mean investors make relatively higher returns in stable, less risky investments. Call options allow these investors exposure to price increases in the riskier stock market without committing to large up-front investments. The greater demand for call options, in turn, increases their price.

Which of the following is accurate regarding the capital market line (CML)? A) The portfolios on the new efficient frontier are some combination of the risk-free asset and the market portfolio. B) The CML is a line that begins at the risk-free rate of return and crosses the efficient frontier at the market portfolio. C) The CML represents the most efficient portfolios of individual stocks. D) The slope of the CML is an investor's indifference curve.

A) The portfolios on the new efficient frontier are some combination of the risk-free asset and the market portfolio. The CML is a combination of the risk-free return and the market portfolio. Option (B) is incorrect, as the CML does not cross the efficient portfolio, rather it is tangent to it. Option (C) is incorrect as the CML consists of the market portfolio and the Rf. Option (D) is incorrect as the slope is the Sharpe Ratio.

A high-income investor owns a portfolio of bonds. Interest rates abruptly and sharply increased. Which of the following strategies would be most helpful in order to take advantage of the decrease in the value of this investor's fixed-income securities? A) The tax swap B) The spread swap C) The bullet strategy D) The barbell strategy

A) The tax swap As interest rates increase, the price of bonds decreases. This drop in value may create an opportunity to recognize the capital loss for tax purposes, which is the purpose of the tax swap.

Which of the following choices correctly orders Treasuries, corporate bonds, and equities from the least chance of a loss of principal to the most chance of a loss of principal? A) Treasuries, corporate bonds, equities B) Treasuries, equities, corporate bonds C) Equities, corporate bonds, Treasuries D) Corporate bonds, Treasuries, equities

A) Treasuries, corporate bonds, equities Corporate bonds have higher default risk than Treasuries, but they offer better principal protection than equities.

A weakness of the buy-and-hold strategy is that it will almost certainly result in A) an asset allocation that looks much different than the original. B) returns far lower than what was expected. C) much higher investment expenses than other strategies. D) far more work on the behalf of the investment manager than other strategies.

A) an asset allocation that looks much different than the original. Buy and hold implies that the investor does not rebalance the portfolio weights even after significant swings in prices. This can result in weights that are substantially different from the beginning-of-period weights and very likely to produce a portfolio with greater volatility than originally intended.

The process of mark-to-market A) posts gains or losses to each account daily and may result in margin calls. B) impacts only short positions. C) occurs only on the contract's expiration date. D) is not required on all futures contracts.

A) posts gains or losses to each account daily and may result in margin calls. Mark-to-market efficiently puts futures contracts on a "pay-as-you-go" basis. It is required for all contracts.

A financial analyst computes the present value of a firm's operating cash flows to calculate an intrinsic value of the shares. The analyst is most likely using A) the discounted cash-flow technique. B) the relative valuation methodology. C) indexing. D) technical analysis.

A) the discounted cash-flow technique. Discounted cash-flow analysis includes estimating future cash flows and then discounting them at an appropriate interest rate. The present value is often referred to as the intrinsic value of the company.

Kay Fields compares the fees and expenses of several equity funds. Fields will likely have the most difficulty in evaluating A) trading costs. B) the 12b-1 fees. C) the back-end loads. D) the front-end loads.

A) trading costs. Trading costs are the most difficult for an investor to assess, partly because the other fees and expenses are disclosed to investors in the prospectus, but also because some trading costs are implicit. Examples of implicit trading costs are bid-ask spreads, market-impact costs, and opportunity costs.

A writer of a put or call may have tax consequences in all the following situations EXCEPT A) when the option premium is received. B) when the option is exercised. C) when the option expires. D) when the option position is closed.

A) when the option premium is received. There are no tax consequences when an option is initially written. Tax consequences only arise when the option is exercised, when the option expires, or when the option position is closed with an offsetting trade.

The Anderson bond is a 5 percent coupon bond with semiannual coupon payments that matures in 10 years. If the YTM for this bond is 4 percent, what is the value of the bond? A) $1,081.11 B) $1,081.76 C) $1,124.35 D) $1,125.03

B) $1,081.76 P/YR 2 FV $1,000.00 N 20 i 4.0% Pmt $25.00 PV$1,081.76

Three years ago, Ty purchased 1,000 shares of Pope Industries, Inc., for $10 per share. He signed an agreement with the company that allowed the company to use his dividend payments to purchase additional shares for him. Over the last 3 years, Ty received a total of $1,200 in dividend payments, which purchased an additional 100 shares of stock. If Ty sells all of his shares for $24,000, what is his taxable gain? A) $0 B) $12,800 C) $14,000 D) $24,000

B) $12,800 Ty's adjusted basis in the shares equals the initial purchase price of the shares, $10,000, plus the dividends of $1,200 that were reinvested, since Ty was required to pay tax on the dividend payments for the year in which they were made. When he sells the shares, he realizes $24,000 less his adjusted basis of $11,200, which equals a taxable gain of $12,800.

Higgins purchased 2,000 shares of Dunlap, Inc., stock for $28 per share three years ago and paid a $50 brokerage commission on the transaction. This year he sold the shares for $37 per share and paid a $50 brokerage commission on the transaction. What is the amount of his taxable gain or loss? A) $0 B) $17,900 C) $18,000 D) $74,000

B) $17,900 The amount realized from the sale is $74,000 − $50 commission = $73,950. His cost basis is $56,000 + $50 commission = $56,050. The taxable gain is $73,950 − $56,050 = $17,900.

Onion, a large technology company, has stock that pays a $1 dividend semiannually. Its current earnings per share is $10. If the stock is currently trading at $120, what is the dividend yield percentage? A) 0.83% B) 1.67% C) 10.00% D) 20.00%

B) 1.67% The dividend yield percentage is equal to the total annual dividends per share divided by the stock price. ($1 × 2) / $120 = 1.67%

Fred bought 100 shares of Apple at $115 per share. One year later, he sold the stock for $152 per share. During the year, Apple declared and paid dividends of $2 per share. What was Fred's holding period return? A) 32% B) 34% C) 73% D) 206%

B) 34% (Net proceeds + dividend - interest) ÷ equity invested = HPR[($152 - $115) + $2] ÷ $115 = 33.91%

Ollie is considering two portfolios: 1) Portfolio A with a return of 10 percent and a standard deviation of 20 percent, and 2) Portfolio B with a return of 6 percent and a standard deviation of 8 percent. Assuming the correlation between A and B is zero and Ollie invests 40 percent in A and 60 percent in B, what is the portfolio standard deviation? A) 7.60% B) 9.33% C) 12.8% D) 14.0%

B) 9.33% Portfolio standard deviation = sqrt[(w1^2 x SD1^2) + (w2^2 x SD2^2)] Portfolio standard deviation = sqrt[(0.4^2 x 20^2) + (0.6^2 x 8^2)] Portfolio standard deviation = sqrt[(0.16 x 400) + (0.36 x 64)] Portfolio standard deviation = sqrt(64 + 23.04) = sqrt(87.04) = 9.33

Juan bought XYZ Company stock at $40 per share. Today, the stock sells for $52 per share. Juan likes the long-term prospects for XYZ stock but wants some protection against price decreases. Which of the following orders is the best way for Juan to both participate in future price increases and ensure a minimum profit of $6 per share? A) A limit order to sell at $46 B) A stop-limit order, stop price = $47, limit price = $46 C) A limit order to buy at $40 D) A stop-sell order, stop price = $46

B) A stop-limit order, stop price = $47, limit price = $46 A limit order will not work as it is a sell order at or above $46 that would be executed today. It does not provide the downside protection Juan is looking for. The stop-limit order will place a limit order with a price of $46 once the stock reaches $47. The stop-sell order results in a market order once the stock price reaches $46, which likely will result in the order being filled below $46.

Which of the following statements regarding collateralized mortgage obligations (CMOs) is correct? A) All tranches receive principal payments throughout the term. B) All tranches receive interest payments throughout the term. C) Tranches with shorter maturities are not subject to prepayment risk. D) Tranches with longer maturities are not subject to default risk.

B) All tranches receive interest payments throughout the term. While interest is paid to all tranches, principal is paid only to the first tranche until that tranche is retired, and then it is paid to the second tranche until it is retired, and so on. All tranches are subject to default risk and prepayment risk.

At 11 PM EST, an investor in New York buys shares of a company listed on the NYSE. Through which of the following did the investor make this purchase? A) An organized exchange B) An over-the-counter market C) The fourth market D) The primary market

B) An over-the-counter market The "third market" involves over-the-counter trades of exchange-listed securities after those exchanges have closed.

Portfolio A has a weighted beta coefficient of 1.5, and Portfolio B has a weighted beta coefficient of 0.9. With these assumptions, which of the following statements is correct? A) Because Portfolio A has a beta greater than 1.2, it would be preferred by rational investors. B) Assuming the market were to drop by 5 percent, Portfolio B should drop less than Portfolio A. C) Neither portfolio is as volatile as the market. D) Portfolio A would be considered a "defensive" portfolio.

B) Assuming the market were to drop by 5 percent, Portfolio B should drop less than Portfolio A. Option (B) is correct. Option (A) is incorrect because there is insufficient information to know whether a rational investor would prefer Portfolio A; we would also need to know the portfolio's return to make this judgement. A positive alpha, on the other hand, would be preferred by rational investors. Options (C) and (D) are incorrect, as Portfolio A is more volatile than the market.

Philip buys undeveloped beachfront property. He is subject to all of the following risks EXCEPT A) Market risk B) Default risk C) Liquidity risk D) Political risk

B) Default risk Philip, as an investor in real estate, is subject to market risk, liquidity risk and political risk. He is not subject to default risk. There is no indication that there is debt involved in this question. In addition, he would not be the one subject to default risk even if debt was involved in the purchase of the real estate.

Identify the financial asset with the highest Treynor ratio if the risk-free rate of interest is 3%. Asset Average Return Standard Deviation Beta R2 Asset Fund 1 Average Return 10% Standard Deviation 13% Beta 0.7 R2 70 Asset Fund 2 Average Return 18% Standard Deviation 22% Beta 1.2 R2 65 Asset Fund 3 Average Return 20% Standard Deviation 30% Beta 1.7 R2 85 Asset Index A Average Return 14% Standard Deviation 17% Beta 1.0 R2 100 A) Fund 1 B) Fund 2 C) Fund 3 D) Index A

B) Fund 2 Fund 1 (10 − 3) ÷ 0.70 = 10 Fund 2 (18 − 3) ÷ 1.2 = 12.5 Fund 3 (20 − 3) ÷ 1.7 = 10 Index A (14 - 3) ÷ 1.0 = 11

Which of the following statements concerning convertible bonds is (are) correct? I. Convertible bonds provide an inexpensive way to buy stock.II. Convertible bonds provide a lower yield than do straight bonds of comparable credit risk and maturity because of the attractiveness of the conversion feature. A) I only B) II only C) Both I and II D) Neither I nor II

B) II only Statement I is incorrect because convertible bonds normally trade at a premium-to-conversion value. Thus, buying a convertible bond and then converting the bond provides an investor with fewer shares of stock than if the investor had purchased the stock outright directly at the start.

Which of the following statements regarding the primary market is (are) correct? I. Investment bankers most commonly use best efforts to facilitate primary offerings.II. Initial public offerings are a significant component of the primary market. A) I only B) II only C) Both I and II D) Neither I nor II

B) II only Statement I is incorrect because investment bankers most commonly use a firm commitment to facilitate a primary offering.

Which of the following statements is/are correct? I. Longer maturity bonds are less volatile than shorter term bonds when interest rates fluctuate. II. Higher coupon bonds are less volatile than lower coupon bonds when interest rates fluctuate. A) I only B) II only C) Both I and II D) Neither I nor II

B) II only When interest rates change, bonds with higher coupon payments and shorter maturities are less volatile.

Which of the following is an index designed to track the performance of large- and mid-cap securities from 21 developed countries? A) Wilshire 21 B) MSCI EAFE C) Barron's 400 D) Euro 21

B) MSCI EAFE The MSCI EAFE index is often used as a benchmark for foreign asset managers.

Security A has the following returns over 4 years: 4%, 7%, 0%, and -1%. What is the mean return and the standard deviation (sample) for Security A? A) Mean of 2.5% and standard deviation of 3.2% B) Mean of 2.5% and standard deviation of 3.7% C) Mean of 4% and standard deviation of 3.2% D) Mean of 4% and standard deviation of 3.7%

B) Mean of 2.5% and standard deviation of 3.7% The mean is 2.5 percent and the standard deviation is close to 3.7 percent. You can calculate these using the ∑+ key on a financial calculator.

An investor buys a call option on ABC stock with an exercise price of $45 for $5 per share. When ABC increases to $50, the call is exercised, and the stock is immediately sold. What is the result? A) A loss of $500 B) No gain or loss C) A profit of $500 D) A profit of $1,000

B) No gain or loss ($5 gain from stock price × 100) − ($5 premium × 100) = $0

A corn farmer who wants to hedge the price of corn should enter into what type of contract? A) Buy a corn futures contract. B) Sell a corn futures contract. C) Long position in a corn call-option contract. D) Long position in a commodity put-option index.

B) Sell a corn futures contract. A corn farmer would enter into a short (sell) futures contract, because he wants to sell his corn.

Which rebalancing strategy incurs the lowest transaction costs? A) The constant-weighting strategy B) The buy-and-hold strategy C) The tactical allocation strategy D) The dynamic allocation strategy

B) The buy-and-hold strategy The buy-and-hold strategy is a strategy that avoids the transaction costs of periodic rebalancing by simply allowing investments to rise or fall in value over time. The primary disadvantage of this strategy is that one's asset allocation will be affected by the past performance of investments held in the portfolio. In order to preserve a desired asset allocation, some other form of rebalancing may be preferable despite the higher transaction costs.

Which of the following is a violation of the wash-sale rule? A) You sell 27 shares of an S&P 500 index fund at a loss. Then 15 days later, you buy 42 shares of a Russell 3000 index fund. B) You have brokerage accounts at two firms. At the first firm, you buy 100 shares of PQR stock. Then 22 days later, you sell 100 shares of PQR stock, bought the prior year, for a loss in your account at the other firm. C) Your spouse buys 200 shares of UK stock. Then 31 days later, you sell 100 shares of the same stock, bought 43 days earlier, at a slight loss. D) You sell 100 shares of UK stock at a slight gain. Then 4 days later, your spouse buys 200 shares of the same stock.

B) You have brokerage accounts at two firms. At the first firm, you buy 100 shares of PQR stock. Then 22 days later, you sell 100 shares of PQR stock, bought the prior year, for a loss in your account at the other firm. Option (A) is incorrect because the two indices are sufficiently different. Option (C) is incorrect because the purchase is more than 30 days prior to the sale. Option (D) is incorrect because the stock is sold at a gain.

Zoe has been saving diligently for many years and has accumulated an appropriate emergency fund along with substantial retirement savings that will allow her to enjoy all of her favorite activities during retirement and still make generous gifts to her nieces and nephews over the next several years. Zoe experiences extreme anxiety when her portfolio declines in value. Which of the following is true? A) Zoe's willingness (tolerance) to take on risk is greater than her ability (capacity) to take on risk, so her portfolio should have a moderately aggressive asset allocation. B) Zoe's ability (capacity) to take on risk is greater than her willingness (tolerance) to take on risk, so her portfolio should have a conservative asset allocation. C) Zoe's willingness (tolerance) to take on risk is greater than her ability (capacity) to take on risk, so her portfolio should have a conservative asset allocation. D) Zoe's ability

B) Zoe's ability (capacity) to take on risk is greater than her willingness (tolerance) to take on risk, so her portfolio should have a conservative asset allocation. Risk capacity is a measurement of the amount of risk a client can afford to take on. Risk tolerance is the amount of risk a client is willing to take on. When risk capacity and risk tolerance are not in alignment, the more conservative of the two should determine the planning recommendations. In this case, Zoe has the ability to take on risk; however, since she lacks the tolerance (willingness) to take on risk, her portfolio should be invested with a more conservative asset allocation.

National Fund is a closed-end fund with a cash balance of $80 million. American Fund is an open-end fund with a cash balance of $300 million. Both National and American have $2 billion in market value of assets. National has less cash most likely because it A) invests in less liquid assets than American. B) does not permit investors to redeem shares at any time. C) has greater management fees than American. D) has less access to capital than American.

B) does not permit investors to redeem shares at any time. Open-end funds must have much more cash on hand than closed-end funds because their shareholders can redeem shares daily, and those transactions occur at the closing net asset value of the fund. Closed-end fund shareholders must sell their shares on an exchange to redeem them, thus requiring closed-end funds to have minimal cash on hand.

For a call option contract, the price at which the option holder can buy the underlying security is called the A) premium. B) exercise price. C) intrinsic value. D) time value.

B) exercise price. The price at which an option holder buys the underlying stock is called the exercise price or the strike price.

Devon Baines is a board member with DDO, Inc., a large pharmaceutical firm. At a recent meeting, the board votes to take over a smaller competitor whose shares trade in the OTC market and are priced at $50 per share. DDO will offer $75 per share, and the offer will become public 10 hours after the meeting. Baines decides to purchase shares in the target firm immediately after the meeting, but when he calls his broker to place the order, he is informed that the price of the target firm is $75. The market is most likely in a(n) A) weak form. B) strong form. C) semi-strong form. D) inefficient form.

B) strong form. A market in which prices reflect all relevant information, including private information, is a strong-form market. In this case, Baines was hoping to purchase at $50 but was only able to place the order when the price had risen to $75 per share, indicating that the price already reflected the new private information about the takeover. This can only occur in a strong-form market.

Cashflow, Inc. expects to generate $200 million in operating cash flows during the next year. It estimates its long-term dividend growth rate to be 3 percent, and it has 100 million shares outstanding. What is the intrinsic value of Cashflow, Inc., if your required rate of return is 9 percent? A) $16.67 B) $22.22 C) $33.33 D) $66.67

C) $33.33 .Intrinsic value = Total market value ÷ Outstanding Shares Total market value = Operating Cash Flow ÷ (Required Rate of Return - Dividend Growth Rate) Total market value = $200,000,000 ÷ (0.09 − 0.03) = $200,000,000 ÷ 0.06 = $3,333,333,333 Intrinsic value = $3,333,333,333 ÷ 100,000,000 = $33.33

Jacques has a convertible bond with a par value of $1,000 that is trading in the market for $925. The bond is convertible into 50 shares of XYZ stock. The current market price of XYZ stock is $17.50 per share. What is the bond's conversion premium? A) $0 B) $76 C) $50 D) $125

C) $50 The bond is convertible into 50 shares, which would be worth $875. The bond is trading at $925. Thus, the conversion premium equals the difference, or $50.

$100 of stock is purchased with a 50% initial margin and a 30% maintenance margin. At what price would a margin call occur? A) $28.58 B) $70.00 C) $71.42 D) $85.00

C) $71.42 The formula used to determine the price below when a margin call will occur is: debt / (1 − MMR). For example, if a 50% initial margin rate and a 30% maintenance margin rate apply, a stock that was purchased when the price was $100 would receive a margin call if the price falls below $50 / (1 − 0.30) = $71.42. The price drop is $28.58, or 1 − debt / (1 − MMR).= 1 − (0.50 / 0.70) = 28.58%.

What is the maximum possible federal tax rate applied to capital gains from the sale of common stock? A) 18.8% B) 20.0% C) 23.8% D) 28.0%

C) 23.8% The maximum possible capital-gains rate is the base 20% maximum plus the 3.8% net investment income tax for a total of 23.8%.

A real estate investor bought a property for $150,000 and rented it out over 3 years before selling it for $160,000. His income and expenses over this period are shown in the table below. Based on this information, what was his dollar-weighted return? Year Income Expenses 1 $7,000, -$10,000 2 $9,000, -$10,000 3 $10,000, -$1,000 A) 1.39% B) 2.40% C) 3.18% D) 3.33%

C) 3.18% First, calculate each time period's total cash inflows and outflows: Period 0 -150,000 Period 1 -3,000 Period 2 -1,000 Period 3 169,000 Then, to solve this problem using an HP10bII+, use the following keystrokes: 150,000 [+/-] [CFj] 3,000 [+/-] [CFj] 1,000 [+/-] [CFj] 169,000 [CFj] [SHIFT] [IRR/YR] The correct answer is 3.1809, or about 3.18%.

Sam's retirement fund is expected to earn a nominal rate of 7 percent, and the inflation rate is estimated at 3 percent. What is Sam's real rate of return? A) 1.43% B) 2.33% C) 3.88% D) 4.00%

C) 3.88% Real return = (1.07 ÷ 1.03) − 1 = 3.8835%

Andy is considering purchasing a 12-year bond that is selling for $1,300. What is the yield to maturity (YTM) for this bond if it has an 8 percent coupon, paid semiannually? A) 4.63% B) 4.68% C) 4.70% D) 4.72%

C) 4.70% P/YR 2 PV ($1,300) N 24 Pmt $40 FV $1,000 i 4.70%

Sam has a $3-million fixed-income portfolio that consists of Bond A, Bond B, Bond C, and Bond D. The bonds have durations of 2, 3, 8, and 10, respectively. If Sam has 20 percent invested in Bond A, 30 percent in Bond B, and 25 percent invested in each of the other two bonds, what is the duration for the portfolio? Assume that the correlation among the bonds is 0.5. A) 5.50 B) 5.75 C) 5.80 D) 6.20

C) 5.80 0.2 × 2) + (0.3 × 3) + (0.25 × 8) + (0.25 × 10) = 0.4 + 0.9 + 2.0 + 2.5 = 5.8

Marleen, who lives in Louisiana, is in the 20 percent federal tax bracket and 5 percent state income tax bracket. Which of the following bonds that she is considering purchasing has the highest after-tax yield? A) A treasury bond paying 4% B) A corporate bond paying 4.5% C) A Louisiana municipal bond paying 3.5% D) A Texas municipal bond paying 3.6%

C) A Louisiana municipal bond paying 3.5% Corporate bonds are subject to federal and state income tax. Treasury bonds are subject to federal income tax only. Municipal bonds are not subject to federal income tax, but they are subject to state income tax if they are not issued by the taxpayer's state of residence. The La. muni has the highest after-tax return.Treasury = 3.200%Corporate = 3.375%La. Bond = 3.500%Tex. Bond = 3.42%

Andrea believes that the economic cycle is at its peak and that a severe downturn is likely in the near future. She would like to invest in a stock that will remain relatively stable if the expected downturn does take place. Which of the following is the most suitable investment to meet her goal? A) A growth stock B) A stock with a beta close to 1 C) A defensive stock D) A stock with a beta higher than 1

C) A defensive stock Defensive stocks are firms that provide products that are necessary for everyday life and are relatively unaffected by general fluctuations in the economy. They typically have betas of less than one.

Which of the following is an advantage of mutual funds over ETFs? A) Lower stock turnover B) Lower average expenses C) Ability to redeem at net-asset value D) Greater tax efficiency

C) Ability to redeem at net-asset value Mutual fund shares can be redeemed for net-asset value while ETFs can be sold at market value, which will be very close to net-asset value if the shares are highly liquid. ETFs generally are more tax efficient because they tend to have lower stock turnover, and on average, have lower expenses.

Mutual fund 12b-1 fees cover which of the following? A) Portfolio management fees B) Administrative costs C) Advertising and marketing fees D) Organizational costs

C) Advertising and marketing fees The 12b-1 fee structure allows for up to 1% allowance for distribution costs, including advertising and promotion.

All of the following statements regarding U.S. Treasury securities are correct EXCEPT: A) Treasury bills (but not Treasury bonds or notes) are considered money market securities. B) Interest income from Treasury securities is tax-free at the state level but not at the federal level. C) All Treasury securities pay interest. D) Treasuries with longer maturities tend to have higher yields.

C) All Treasury securities pay interest. Treasury bills do not pay interest - they are pure discount instruments.

Which of the following advisors must register with the U.S. Securities and Exchange Commission (SEC)? A) An advisor who sells insurance products B) An advisor who is registered with his or her state securities regulator C) An advisor who manages over $100 million in assets D) An advisor who declines to be regulated by FINRA

C) An advisor who manages over $100 million in assets Advisors with at least $100 million in assets under management must register with the SEC. Advisors with less than $100 million in assets under management must register with their respective state regulator.

The decision of whether to go public is an important one for private companies. Which of the following statements is (are) a disadvantage of going public? Loss of some control over business operations The obligation to file annual and quarterly reports to the SEC A) I only B) II only C) Both I and II D) Neither I nor II

C) Both I and II Companies that go public have less control over their business operations and must file reports to the SEC.

Which of the following bonds are most likely to be called? A) Treasury bonds after interest rates decreased. B) Treasury bonds after interest rates increased. C) Coupon bonds after interest rates decreased. D) Coupon bonds after interest rates increased.

C) Coupon bonds after interest rates decreased. Bonds are called when a company can refinance its debts at a lower interest rate. Treasury bonds are noncallable.

All the following statements regarding investment strategies for tax-advantaged accounts are correct EXCEPT: A) The general rule is that one should place securities with interest income in qualified accounts and securities with good prospects for capital gains in nonqualified accounts. B) The major tax-advantaged accounts include IRAs and qualified plans through work such as the 401(k). C) Investors should always maximize their tax-advantaged accounts before contributing to taxable brokerage accounts. D) Tax-advantaged accounts allow investors to defer the recognition of taxable income or even experience tax-free growth.

C) Investors should always maximize their tax-advantaged accounts before contributing to taxable brokerage accounts. There are cases in which investors should prioritize contributing to taxable brokerage accounts (e.g., when creating an emergency fund or a down payment fund). Tax-advantaged accounts generally limit withdrawals, which may subject the investor to additional taxes or penalties if the funds are used for certain nonqualified purposes.

Which of the following investing risks is most significant for real estate investors as compared to investors in publicly traded stocks? A) Market risk B) Default risk C) Liquidity risk D) Idiosyncratic risk

C) Liquidity risk Liquidity risk is far higher in real estate investing than in trading in stock that can be sold on a secondary market exchange. Stock investors are subject to market risk, idiosyncratic risk, and the risk that the firm will declare bankruptcy.

A small-cap fund manager invests in a technology company that grows and eventually becomes part of the S&P 500. What problem does the fund manager face by not selling shares of this company? A) Long-term capital gains B) Liquidity C) Style shift D) Low Jensen's alpha

C) Style shift When a small-cap fund successfully identifies firms that grow in value and become large firms, they need to sell shares of these firms to maintain their small-cap style orientation.

All of the following regarding TIPS are true EXCEPT: A) The interest paid on TIPS changes based on inflation and the changing principal amount of the TIPS. B) TIPS are considered low-risk investments. C) TIPS are most appropriate for young investors with many decades until retirement. D) If deflation occurs over the life of the TIPS, the investor receives the greater of the principal amount or the par value.

C) TIPS are most appropriate for young investors with many decades until retirement. TIPS are a low-risk, low-reward investment that only pays well in periods of high inflation. Young people are less exposed to inflation risk because their incomes and higher-risk investments will help keep pace with inflation. Older people into retirement, on the other hand, are very exposed to inflation risk, making TIPS a more appropriate investment for them.

Which of the following techniques or strategies would take advantage of a perceived undervaluation in the energy sector of the economy? A) Dollar-cost averaging into a portfolio B) Strategic asset allocation rebalancing C) Tactical asset allocation D) Index funds

C) Tactical asset allocation Tactical asset allocation is in response to short-term market conditions. Dollar-cost averaging is a method of investing over time. Strategic asset allocation is a long-term strategy, not one that takes advantage of sector imbalances.

If an investor owns a single share of a stock with a beta of 0.75, what can you conclude about his investment risk relative to the market? A) The investor's total risk is 3/4 of the risk of the market. B) The investor is taking 75 percent more total risk than the risk of the market. C) The investor's systematic risk is ³⁄₄ of the risk of the market. D) The investor's diversifiable risk is 75% more than the total risk of the market.

C) The investor's systematic risk is ³⁄₄ of the risk of the market. Beta only measures systematic risk relative to 1, the market's beta. Beta assumes a stock is added to an already diversified portfolio. Because a single stock has both diversifiable as well as systematic risk, an investor who purchases only a single stock may have a total risk greater than the market's risk.

Byte, Inc., had net earnings of $2.5 billion last year. It has 500 million shares of common stock outstanding, which stayed the same since last year. It paid 50 cents per share per quarter this year as a dividend payment, which is the same as last year. This year's earnings are 20% higher than last year. Which of the following is correct for this year? A) The retention ratio equals 33.33%. B) The retention ratio equals 50.00%. C) The payout ratio equals 33.33%. D) The payout ratio equals 40.00%.

C) The payout ratio equals 33.33%. The dividend per share equals $2.00. The EPS equals $5.00 last year, which is found by dividing net earnings by outstanding shares. This year's EPS is 20% higher or $6.00.The payout ratio = dividend / EPS $2.00 / $6.00 = 33.33%

Which of the following can be greatly reduced by diversification? A) Systematic risk B) Market risk C) Unsystematic risk D) Systematic and unsystematic risk

C) Unsystematic risk Unsystematic risk is reduced by diversification.

Most variation in portfolio returns over an extended period of time is attributable to A) security selection. B) market timing. C) asset allocation. D) market cycles.

C) asset allocation. Over time, asset allocation represents the largest impact on performance for investors.

A stock market participant that buys shares of stock in a secondary market and holds shares as part of its inventory is most likely a(n) A) stock exchange. B) independent advisor. C) broker-dealer. D) self-regulatory organization.

C) broker-dealer. Broker dealers serve two functions: to act as an intermediary in helping clients buy and sell securities and to generate a profit on the trading of undervalued securities. In their dealer function, broker-dealers can hold inventory when they believe the shares are undervalued with the intent of selling them at a later date when the price rises.

Private equity is regularly characterized by A) the ability to accurately estimate the beta of a target business. B) low funding risk. C) high liquidity risk. D) active secondary markets to trade private equity positions.

C) high liquidity risk. It may be difficult for a private equity investor to sell her position. Statement (A) is incorrect because it is very difficult for private equity firms to gain enough access to relevant financial information of target businesses to be able to accurately estimate the target's beta. Statements (B) and (D) are incorrect because PE has significant funding risk at least some of the time, and there is a limited secondary market.

Harold would like to purchase shares of a large, established company. He will most likely make his purchase A) in the primary market. B) from an underwriter. C) in the secondary market. D) from the issuing corporation.

C) in the secondary market. Newly issued shares are sold in the primary market, with the assistance of an underwriter. The purchase and sale of securities that have already passed through the primary market occurs in the secondary market. The shares of a large, established company have likely been through the primary market many years ago and are currently trading in the secondary market.

An "in the money put" A) has an intrinsic value < 0. B) is where the stock price > strike price. C) is where the stock price < strike price. D) has a time value > 0.

C) is where the stock price < strike price. The stock price is less than the strike price.

An investment manager will adopt an active management strategy in an effort to A) reduce costs. B) reduce workload. C) outperform market returns. D) mirror market returns.

C) outperform market returns. The active approach suggests that investment managers can outperform the market over long periods of time.

A venture capitalist provides EUR 25 million to a business that has developed an innovative fishing product. Distribution channels have not yet been identified, although some revenues have been generated. The VC has most likely entered the process at the A) bridge stage. B) initial public offering stage. C) startup stage. D) expansion stage.

C) startup stage. The startup stage is characterized by businesses that have a product to sell but need help with marketing, packaging, and distribution. The VC frequently acts as an expert in this stage by providing advice and consulting services. The startup stage can produce some revenues, but the business lacks mass production capabilities.

An individual seeking diversification by including alternative investments in her portfolio is most likely to benefit if A) there are tax advantages associated with the alternative investment. B) the alternative investment generates high returns. C) there are low correlations between her portfolio and the alternative investment. D) the alternative investment is a good hedge against inflation.

C) there are low correlations between her portfolio and the alternative investment. Diversification only works if there is less than a perfect +1.0 correlation between a portfolio and the alternative investment. The primary reason for the popularity of alternatives is that they tend to have low or even negative correlations with traditional securities.

Cody purchased 400 shares of NAY stock six years ago when it was trading at $65 per share. Unfortunately, NAY has been steadily declining. Cody sold his shares this year for $18 per share. This year Cody also sold 800 shares of a mutual fund that he purchased 6 months ago. His average cost per share was $15 and he sold the shares for $32. Assuming Cody had no other capital transactions this year, what is Cody's tax consequence from these transactions? A) $5,200 long-term loss this year B) $18,800 long-term loss, $13,600 short-term gain taxed at ordinary income rates this year C) $3,000 ordinary loss this year with a $15,800 ordinary loss carried forward up to 5 years D) $3,000 ordinary loss this year with a $2,200 loss carried forward indefinitely

D) $3,000 ordinary loss this year with a $2,200 loss carried forward indefinitely The NAY stock sale resulted in a long-term loss of $18,800 ($47 loss per share × 400 shares). The mutual fund sale resulted in a short-term gain of $13,600 ($17 gain × 800 shares). The entire $13,600 gain is offset by the loss, leaving a net long-term loss of $5,200. Cody can take $3,000 of the loss against ordinary income this year, and the remaining loss is carried forward indefinitely (until death).

Austin invested in the Very Value mutual fund 5 years ago. His returns were 25%, -5%, 10%, 0%, and 50%, respectively. What is the difference between the arithmetic average and the geometric average return over the 5 years? A) 0% B) 1.1% C) 1.3% D) 1.6%

D) 1.6% Arithmetic average = 16% Geometric average = [(1 + 25%) × (1 + -5%) × (1 + 10%) × (1 + 0%) × (1 + 50%)](1/ 5) - 1 = 14.4% Difference = 16% − 14.4% = 1.6%

Parker, who lives in Covington, Louisiana, purchased three bonds from a company based in Brazil that were yielding 9.75 percent and paid a 12 percent coupon semi-annually. The company went bankrupt, and Parker never received his money. What type of risk was he subject to when he purchased the bond? A) Interest-rate risk B) Default risk C) Exchange-rate risk D) All of the above

D) All of the above The Brazilian bond is subject to exchange-rate risk, default risk, and interest-rate risk.

Which of the following statements regarding the tax treatment of municipal bonds is correct? A) The 3.8% Net Investment Income Tax (NIIT) applies to the coupon payments received from municipal bonds. B) Public purpose municipal bonds are subject to the alternative minimum tax (AMT). C) Gains from the sale of municipal bonds are tax-free at the federal level. D) Coupon payments from municipal bonds are generally tax-free at the federal level.

D) Coupon payments from municipal bonds are generally tax-free at the federal level. Unlike corporate bonds, whose coupon payments are taxed as ordinary interest income, the coupon payments from municipal bonds are not subject to tax under the regular tax system and are not subject to the NIIT. If the taxpayer is subject to AMT, the coupon payments from private activity municipal bonds are taxable; however, the coupon payments from public purpose municipal bonds remain tax-free. Gains from the sale of municipal bonds are taxable gains.

All of the following statements are correct EXCEPT: A) Yields on T-Bills are typically lower than commercial paper since there is less risk. B) Negotiable CDs have very large balances and are tradeable in the secondary market. C) Commercial paper is not considered default risk-free. D) Interest income and capital gains from Treasury securities are both exempt from state and local taxes.

D) Interest income and capital gains from Treasury securities are both exempt from state and local taxes. T-bills are default risk-free, while commercial paper has some, although minor, default risk. Interest income is exempt from state and local taxes, but capital gains are normally taxable at the state and local level.

Assume that the 1-, 2-, 3-, 5-, 10-, 20-, and 30-year rates were 7 percent, 6.5 percent, 6 percent, 5.4 percent, 5.2 percent, 5.0 percent, and 4.8 percent, respectively. What type of yield curve is this? A) Humped B) Normal C) Flat D) Inverted

D) Inverted The yield curve is downward sloping, which is known as inverted.

All of the following are correct regarding derivative securities EXCEPT: A) Derivatives can be used for speculation and hedging. B) The use of leverage is both an advantage and a disadvantage of derivatives. C) The value of a derivative is tied to the value of an underlying security or asset. D) Most types of derivatives are suitable investments for unsophisticated investors.

D) Most types of derivatives are suitable investments for unsophisticated investors. The use of leverage exaggerates both gains and losses, which can be an advantage or a disadvantage. This exaggeration of gains and losses, along with a derivative's relative complexity, makes most types of derivatives unsuitable for small, unsophisticated investors.

Which of the following is (are) correct with respect to bond portfolio strategies? I. A portfolio that has 40% invested in bonds with a 2-year maturity and the other 60% invested in bonds with a 10-year maturity is known as a bullet portfolio. II. If the portfolio's duration statistic matches the intended holding period, then the portfolio is always referred to as a laddered portfolio. A) I only B) II only C) Both I and II D) Neither I nor II

D) Neither I nor II The portfolio in Statement I is an example of a barbell portfolio. The portfolio in Statement II is an example of an immunized portfolio.

Maggie considers four potential securities with identical expected returns but different correlations with her existing portfolio as part of her asset-selection decision. Which security would provide Maggie with the greatest diversification benefit? A) Security 1 with a correlation coefficient of +1.0 B) Security 2 with a correlation coefficient of +0.32 C) Security 3 with a correlation coefficient of 0.00 D) Security 4 with a correlation coefficient of -0.38

D) Security 4 with a correlation coefficient of -0.38 Since the returns of the four potential securities are the same, the primary consideration is the risk element. The best security is the one with the lowest correlation, which is Security 4.

The return from the CAPM is which of the following? A) The risk-free return B) The actual return C) The risk-premium D) The expected return

D) The expected return CAPM = Rf + β(Rm - Rf ) = the expected return based on beta, the market premium, and the risk-free rate of return

Which is the least likely motivation for investing in mutual funds? A) To seek diversification benefits B) To obtain professional management of investments C) To invest conveniently D) To obtain returns that normally exceed market returns

D) To obtain returns that normally exceed market returns Mutual funds offer diversification, convenience, and professional management, but they frequently perform well below the market return.

An asset-allocation strategy that periodically purchases assets that have fallen in value and sells assets that have risen in value is A) a buy-and-hold strategy. B) a tactical allocation strategy. C) a dynamic allocation strategy. D) a constant-weighting strategy.

D) a constant-weighting strategy. A constant-weighting strategy returns the weights to their original strategic values. Consequently, when the price of a security falls, its weights also fall. The investor is forced to purchase additional shares to increase the allocation.

All of the following are likely to be classified as investment advisers by the U.S. Securities and Exchange Commission (SEC) EXCEPT A) a financial analyst who issues a new report on the performance of value stocks directly to a client. B) an analyst who recommends a triple-A-rated bond to a client. C) a money manager who makes asset allocation assignments for institutional investors. D) a financial firm that acts as a dealer in investment grade bonds.

D) a financial firm that acts as a dealer in investment grade bonds. Broker-dealers are explicitly excluded from being classified as advisers under the legal framework of the Investment Advisers Act. Individuals or firms that issue reports or offer investment advice, including asset-allocation decisions, are considered to fit the definition of an investment adviser.

An investor living in Pennsylvania who buys a municipal bond issued by the state of Wyoming will most likely A) pay federal taxes on the coupon payments. B) have less risk than with a Treasury security with a similar maturity date. C) earn a much higher yield to maturity than similar-risk corporate bonds. D) have a Pennsylvania tax liability on the interest received on the Wyoming bond.

D) have a Pennsylvania tax liability on the interest received on the Wyoming bond. Municipal bonds have tax advantages not enjoyed by corporate bonds, which makes their yields much less than those on corporate bonds. In addition, the interest received on municipal debt is federal tax-free and even state tax-free if the bondholders reside in the state of the issuer. In this case, the Pa. bondholder will have to report the income received on the Wyoming bond and will incur a state income tax liability.

The intersection of the security market line (SML) and the y axis occurs at the A) market premium. B) market portfolio. C) real rate of return. D) risk-free rate of return.

D) risk-free rate of return. The SML is a line that connects the risk-free rate of return, which is on the y-axis, to the market portfolio.

A contract that is an agreement for immediate exchange of funds for assets is called a A) futures contract. B) forward contract. C) call option contract. D) spot contract.

D) spot contract. A spot contract is a contract that calls for immediate delivery of goods. The spot price or rate is the current price or rate.

All of the following positions are examples of the types of holdings found in a hedge fund manager's portfolio EXCEPT A) the long-only position in a gasoline futures contract. B) a put option written on a barrel of oil. C) long and short positions in the equity of two industry leaders. D) the long position only in a highly rated municipal bond.

D) the long position only in a highly rated municipal bond. Hedge funds rarely invest in investment grade municipal bonds, because the risk is generally very low and the return is not sufficient to warrant interest by the hedge fund manager. Hedge funds often invest in options and futures contracts and also try to benefit when one industry leader outperforms its main rival.

All of the following is (are) among the responsibilities of FINRA EXCEPT A) educating investors. B) fostering market transparency. C) writing and enforcing rules governing the activities of all registered broker-dealer firms and registered brokers in the U.S. D) writing and enforcing rules governing the activities of all registered investment advisors.

D) writing and enforcing rules governing the activities of all registered investment advisors. The SEC and state securities regulators are responsible for regulation of registered investment advisors.


संबंधित स्टडी सेट्स

Chemistry Energy Changes and Rates of Reaction Expectations

View Set

Unit 7: Gerontological Care Issues

View Set

Chapter 49 Management of Patients with Urinary Disorders

View Set

Section 9: Other Real Estate Matters in Texas

View Set

PSYCH 303- Research Methods Final (Exam 5)

View Set

Computer Concepts and Applications Test #2

View Set